Đến nội dung

iloveyouproht nội dung

Có 153 mục bởi iloveyouproht (Tìm giới hạn từ 10-06-2020)



Sắp theo                Sắp xếp  

#590300 Tìm giá trị lớn nhất : $a^2+b^2+c^2$

Đã gửi bởi iloveyouproht on 22-09-2015 - 18:34 trong Phương trình, hệ phương trình và bất phương trình

1.Cho $0 \leq  a,b,c \leq 1$ và $a+b+c=7$. Tìm giá trị lớn nhất : $a^2+b^2+c^2$

2.Tìm $x$ : $x^4 + \sqrt{(x^2+1999)} = 1999$

 




#647998 Chứng minh rằng $\sum \frac{a^{2}}{b+...

Đã gửi bởi iloveyouproht on 04-08-2016 - 23:23 trong Bất đẳng thức và cực trị

Cho các số dương a, b, c. Chứng minh rằng:

$\frac{a^{2}}{a+b}+\frac{b^{2}}{b+c}+\frac{c^{2}}{c+a}+\frac{1}{2}(\sqrt{ab}+\sqrt{bc}+\sqrt{ac})\geq a+b+c$

Mình có cảm giác đề bài này sai sai. Mọi người giúp mình với

Cách khác nếu b cần . Gần giống cách của anh Sơn

Ta có

$\frac{a^{2}}{a+b}-a = \frac{-ab}{a+b}$

=> Ta cần cm :

$\sum \frac{\sqrt{ab}}{2} -\sum \frac{ab}{a+b} \geq 0$

Mà : 

$\sum \frac{\sqrt{ab}}{2} -\sum \frac{ab}{a+b} \geq \frac{\sqrt{ab}}{2}-\frac{\sqrt{ab}}{2}=0$( cauchy )

=> ĐPCM




#647822 Chứng minh: 1. $\sum \frac{a}{\sqrt{2...

Đã gửi bởi iloveyouproht on 03-08-2016 - 20:23 trong Bất đẳng thức và cực trị

a, b ,c là 3 cạnh tam giác. Chứng minh:

1) $\frac{a}{\sqrt{2b^{2}+2c^{2}-a^{2}}}+\frac{b}{\sqrt{2a^{2}+2c^{2}-a^{2}}}+\frac{c}{\sqrt{2a^{2}+2b^{2}-c^{2}}}\geq \sqrt{3}$

2) $ab+bc+ca\geq \frac{1}{5}(\sqrt{(2b^{2}+2c^{2}-a^{2})(2a^{2}+2c^{2}-b^{2}})+\sqrt{(2a^{2}+2c^{2}-b^{2})(2a^{2}+2b^{2}-c^{2})}+\sqrt{(2a^{2}+2b^{2}-c^{2})(2b^{2}+2c^{2}-a^{2})})$

Chỗ này đề sai !




#656460 Cho a,b,c dương chứng minh:

Đã gửi bởi iloveyouproht on 02-10-2016 - 20:13 trong Bất đẳng thức và cực trị

Cho a,b,c dương chúng minh:

$\sqrt{\frac{a}{c+b+2a}}+\sqrt{\frac{b}{c+a+2b}}+\sqrt{\frac{c}{a+b+2c}}\leq \frac{3}{2}$

Cách Khác : $(\sqrt{\frac{a}{c+b+2a}}+\sqrt{\frac{b}{c+a+2b}}+\sqrt{\frac{c}{a+b+2c}})^{2}\leq 3 (\sum \frac{a}{c+b+2a})=3(3-\sum \frac{a+b+c}{c+b+2a})=3\left [3-(a+b+c)(\sum \frac{1}{c+b+2a}) \right ]\leq 3\left [ 3-(a+b+c)(\frac{9}{4(a+b+c)}) \right ]^{2}=\frac{9}{4}$

=> Đpcm 

Dấu = xảy ra khi a=b=c




#659627 $A=\frac{a}{\sqrt{b}}+\frac...

Đã gửi bởi iloveyouproht on 27-10-2016 - 22:04 trong Bất đẳng thức và cực trị

1.Tìm giá trị nhỏ nhất của $A=\frac{a}{\sqrt{b}}+\frac{b}{\sqrt{c}}+\frac{c}{\sqrt{a}}$ biết $a+b+c\geq 3$

2.Cho a,b,c dương thuộc $\begin{bmatrix} 3;5 \end{bmatrix}$*

3.Cho a,b,c>0.tìm gtln của $P=\sum \frac{\sqrt{ab}}{c+2\sqrt{ab}}$

4.Tìm min max của $A=\frac{x-y}{x^4+y^4+6}$

bài 3 nha <3

Ta có : $P=\sum \frac{\sqrt{ab}}{c+2\sqrt{ab}} => 2P=\sum \frac{2\sqrt{ab}}{c+2\sqrt{ab}}=3-\sum \frac{c}{c+2\sqrt{ab}}\leq 3-\sum \frac{c}{a+b+c}=2$

=> Max P=1

Dấu = xảy  ra khi a=b=c <3




#687242 $\sum \frac{a}{\sqrt{(1+a^{3...

Đã gửi bởi iloveyouproht on 11-07-2017 - 16:30 trong Bất đẳng thức và cực trị

Cho a, b, c >0 thỏa abc=8 . CMR : $\sum \frac{a}{\sqrt{(1+a^{3})(1+b^{3})}}\geq \frac{2}{3}$

Bđt sai vs đ c=0,02 còn a=b=20 Bài này b chắc chế từ :

 

Cho a, b, c >0 thỏa abc=8 . CMR : $\sum \frac{a^{2}}{\sqrt{(1+a^{3})(1+b^{3})}}\geq \frac{4}{3}$  Đúng không ?




#676229 $\frac{a}{a+bc}+\frac{b}{b+...

Đã gửi bởi iloveyouproht on 04-04-2017 - 20:48 trong Bất đẳng thức và cực trị

Cho $a,b,c>0$ và $a+b+c=1$.Chứng minh

     $\frac{a}{a+bc}+\frac{b}{b+ca}+\frac{c}{c+ab}\leq \frac{9}{4}$

P=$\sum \frac{a}{a+bc}=\sum \frac{a}{a(\sum a)+bc}=\sum \frac{a}{(a+b)(a+c)}=\frac{2(\sum ab)}{\prod (a+b)}$

Ấp dụng bđt : $\prod (a+b)\geq \frac{8}{9}(\sum a)(\sum ab)$

=> P$\leq \frac{2(\sum ab)}{\frac{8}{9}(\sum a)(\sum ab)}= \frac{9}{4}$(đpcm)




#647819 Chứng minh: 1. $\sum \frac{a}{\sqrt{2...

Đã gửi bởi iloveyouproht on 03-08-2016 - 20:18 trong Bất đẳng thức và cực trị

B1 có trong sách STBĐT nên mk lười gõ nữa nha :v

of38fla.png




#648980 $P=\frac{a}{a^{2}+b^{3}}+...

Đã gửi bởi iloveyouproht on 11-08-2016 - 02:46 trong Bất đẳng thức và cực trị

 

Cho các số thực không âm $a,b,c$ và $a+b+c=3$. Tìm giá trị nhỏ nhất của:
$P=\frac{a}{a^{2}+b^{3}}+\frac{b}{b^{2}+c^{3}}+ \frac{c}{c^{2}+a^{3}}$

 

Ta có : $\sum \frac{a^{3}}{a^{2}(a^{2}+b^{3})}(\sum a^{2})(\sum a^{2}+b^{3})\geq (a+b+c)^{3}=27(holder)$

Mà : $\sum \frac{a^{3}}{a^{2}(a^{2}+b^{3})}(\sum a^{2})(\sum a^{2}+b^{3})\geq \sum \frac{a^{3}}{a^{2}(a^{2}+b^{3})}(\frac{(a+b+c)^{2}}{3})(\frac{(a+b+c)^{2}}{3} + (a+b+c)^{3}-3(a+b)(b+c)(c+a)\geq 3\sum \frac{a^{3}}{a^{2}(a^{2}+b^{3})} (30-\frac{8(a+b+c)^{3}}{9})\geq 18\sum \frac{a^{3}}{a^{2}(a^{2}+b^{3})} \geq 27$ ( đã cm trên )

=> Min : $\sum \frac{a^{3}}{a^{2}(a^{2}+b^{3})}$ = $\frac{3}{2}$

Dấu = xảy ra khi a=b=c=1 




#633505 Chứng minh : $\frac{1}{a+b+c} \geq \frac{1}{3} +(1-a)(1-b...

Đã gửi bởi iloveyouproht on 16-05-2016 - 18:20 trong Bất đẳng thức và cực trị

cho $0< a,b,c\leq 1$. Chứng minh : $\frac{1}{a+b+c} \geq \frac{1}{3} +(1-a)(1-b)(1-c)$




#590426 Tìm giá trị lớn nhất : $a^2+b^2+c^2$

Đã gửi bởi iloveyouproht on 23-09-2015 - 12:18 trong Phương trình, hệ phương trình và bất phương trình

1. Từ giả thiết ta có:
$(a-1)(b-1)(c-1)-abc\leq 0\Leftrightarrow abc-(ab+bc+ca)+a+b+c-abc-1\leq 0$
$\Rightarrow 2(a+b+c)\leq 2(ab+bc+ca)+2\Leftrightarrow a^2+b^2+c^2\leq (a+b+c)^2-2(a+b+c)+2$
Đến đây thay số rồi tính
P/S: $a+b+c=7$ không thể xảy ra, bạn hãy sửa lại đề

= 2 b ạ !! Mình nhầm




#637712 Tim Max : $\frac{1}{a^{5}+b^{5}+ab} + \frac{1}{b^{5}+c^{5}+bc}...

Đã gửi bởi iloveyouproht on 02-06-2016 - 23:18 trong Bất đẳng thức và cực trị

Cho a,b,c>0 ; abc=1 .Tim Max : $\frac{1}{a^{5}+b^{5}+ab} + \frac{1}{b^{5}+c^{5}+bc} + \frac{1}{c^{5}+a^{5}+ac}$




#637522 Chứng minh $\sum \frac{1}{a(a+b)} \ge...

Đã gửi bởi iloveyouproht on 02-06-2016 - 00:29 trong Bất đẳng thức và cực trị

Cho a,b,c là các số dương , Chứng minh :  $\frac{1}{a(a+b)} + \frac{1}{b(b+c)} + \frac{1}{c(c+a)} \geq \frac{27}{2(a+b+c)^{2}}$

 




#646738 $(a+b+c)^{3}\geq 7(a+b+c) + \frac{2}{...

Đã gửi bởi iloveyouproht on 27-07-2016 - 16:05 trong Bất đẳng thức và cực trị

Cho a,b,c dương . abc=3 . Chứng Minh : $(a+b+c)^{3}\geq 7(a+b+c) + \frac{2}{3}(a+b+c)^{2}$




#646755 $\geq 1 . Tìm max : \frac{(1+x)(1+y)(1+z)}{1+xy...

Đã gửi bởi iloveyouproht on 27-07-2016 - 16:52 trong Bất đẳng thức và cực trị

.




#676312 $\frac{a^{2}}{a^{2}+ab+b^{2}}+\frac{b^{2}}{b^{2}+bc+c^{2}}+...

Đã gửi bởi iloveyouproht on 05-04-2017 - 20:15 trong Bất đẳng thức và cực trị

Cho a, b, c dương thỏa a+b+c=3. CMR : $\frac{a^{2}}{a^{2}+ab+b^{2}}+\frac{b^{2}}{b^{2}+bc+c^{2}}+\frac{c^{2}}{c^{2}+ca+a^{2}}\geq 1$

Ta có : P=$\sum \frac{a^{2}}{a^{2}+bc+c^{2}}=\sum \frac{1}{1+\frac{b}{a}+\frac{b^{2}}{a^{2}}}$

Đặt : $\left ( \frac{b}{a};\frac{c}{b} ;\frac{a}{c}\right )->\left ( x;y;z \right )$

=>xyz=1

P=$\frac{1}{x^{2}+x+1}$

Tiếp tục đặt : $\left ( x;y;z \right )->\left ( \frac{np}{m^{2}};\frac{nm}{p^{2}};\frac{pm}{n^{2}} \right )$

Ta đưa bđt về cần cm : $\sum \frac{m^{4}}{m^{4}+m^{2}np+n^{2}p^{2}}$

Mà : $\sum \frac{m^{4}}{m^{4}+m^{2}np+n^{2}p^{2}}\geq \frac{(\sum m^{2})^{2}}{\sum m^{4}+\sum m^{2}np+\sum n^{2}p^{2}}$

BĐT sẽ được CM nếu chỉ ra : $(\sum m^{2})^{2}\geq \sum m^{4}+\sum m^{2}np+\sum n^{2}p^{2}$

Hay : $\sum m^{2}n^{2}\geq mnp(\sum m)$(bất đẳng thức này đúng theo cauchy)




#686638 Tuyển tập tính chất trong toán hình học phẳng

Đã gửi bởi iloveyouproht on 05-07-2017 - 22:11 trong Chuyên đề toán THCS

Tài liệu này dc úp trên trang lttk mà giá mua là 50k khá chát. Nên mình đóng thành pdf  để mọi người in và dùng free :)). Nhớ like và share nếu thấy tài liệu bổ ích :)

https://drive.google...aXZrbmlHYXkxVVk

tks thím <3




#667972 CMR: $\sum \frac{1}{3a+3b+2c}\leq...

Đã gửi bởi iloveyouproht on 11-01-2017 - 10:40 trong Bất đẳng thức và cực trị

Cho a, b, c>0 thỏa mãn: $\frac{1}{a+b}+\frac{1}{b+c}+\frac{1}{c+a}=6$

CMR: $\sum \frac{1}{3a+3b+2c}\leq \frac{3}{2}$

Đặt a+b=x;b+c=y;c+a=z ta có : $\sum \frac{1}{x}=6$

Mà : VT=$\sum \frac{1}{2x+y+z}\leq \frac{1}{16}(\sum \frac{4}{x})=\frac{1}{4}(\sum \frac{1}{x})=\frac{3}{2}$ ( đpcm )




#676324 $\frac{a^{2}}{a^{2}+ab+b^{2}}+\frac{b^{2}}{b^{2}+bc+c^{2}}+...

Đã gửi bởi iloveyouproht on 05-04-2017 - 21:02 trong Bất đẳng thức và cực trị

e nghĩ ko dc đặt ntn.Điều kiện bài toán sẽ bị thay đổi,

đặt được bình thường b à :) Vì khi mình đặt thì chỉ ở lần 2 mới liên quan đến diều kiện bài toán b ạ :)




#667915 $\sum \frac{a}{a+b^{2}}\leq...

Đã gửi bởi iloveyouproht on 10-01-2017 - 20:10 trong Bất đẳng thức và cực trị

Cái này hình như phải là $a$ chứ 

$\sum \frac{1}{a+b+c+\frac{b^{2}}{a}}\leq \sum  \frac{1}{2\sqrt{a+\frac{b^{2}}{a}}+b+c}$




#651578 Cho a,b,c là các số thực dương thỏa mãn $a+b+c+2=abc$. Chứng minh r...

Đã gửi bởi iloveyouproht on 27-08-2016 - 23:59 trong Bất đẳng thức và cực trị

Giải:

GT $\Rightarrow$ Tồn tại các số $x,y,z>0$ sao cho $a=\frac{y+z}{x};b=\frac{z+x}{y};c=\frac{x+y}{z}$

BĐT cần chứng minh $\Leftrightarrow \sqrt{\frac{x}{x+y+z}}+\sqrt{\frac{y}{x+y+z}}+\sqrt{\frac{z}{x+y+z}}\leqslant \sqrt{3}$

Đến đây áp dụng BĐT Bunhiacopxki là ra

Dấu "=" xảy ra khi $a=b=c=2$

cái này là sao ạ :D




#646640 $\frac{a}{b} + \frac{b}{c...

Đã gửi bởi iloveyouproht on 26-07-2016 - 21:58 trong Bất đẳng thức và cực trị

Cho a,b,c >0 . a+b+c=3 Chứng minh : $\frac{a}{b} + \frac{b}{c} + \frac{c}{a} + 9abc \geq 9$




#647103 $\frac{a}{b} + \frac{b}{c...

Đã gửi bởi iloveyouproht on 29-07-2016 - 21:38 trong Bất đẳng thức và cực trị

Bạn có thể xem lại vế phải được không, dấu bằng xảy ra khi nào vậy?

Mình cũng k biết nữa :D K biết làm nên up lên cho m.n giải hộ thôi :P




#667880 $\sum \frac{a}{a+b^{2}}\leq...

Đã gửi bởi iloveyouproht on 10-01-2017 - 15:42 trong Bất đẳng thức và cực trị

Cho $a,b,c>0$ thỏa $a+b+c=1$. CM

$\frac{a}{a+b^{2}}+\frac{b}{b+c^{2}}+\frac{c}{c+a^{2}}\leq \frac{1}{4}(\frac{1}{a}+\frac{1}{b}+\frac{1}{c})$

Ta có :

$\sum \frac{a}{a+b^{2}}=\sum \frac{1}{1+\frac{b^{2}}{a}}=\sum \frac{1}{a+b+c+\frac{b^{2}}{a}}\leq \sum \frac{1}{2b+b+c}\leq \frac{1}{16}(\sum \frac{4}{a})=\frac{1}{4}(\sum \frac{1}{a})$=) đpcm




#650311 $(a^{2} + 2)(b^{2} + 2)(c^{2} + 2)\ge...

Đã gửi bởi iloveyouproht on 19-08-2016 - 07:53 trong Bất đẳng thức và cực trị

Cho a, b, c là các số thực không âm. Chứng minh rằng

$(a^{2} + 2)(b^{2} + 2)(c^{2} + 2)\geq 9(ab + bc + ca)$

Đã có ở đây : 

http://diendantoanho...c2-geq-9abacbc/